NF ISUSAu N4 IC

You might also like

Download as pdf
Download as pdf
You are on page 1of 319
Viva Voce in ANATOMY AINOLVNY "22a NA THIRD EDITION ae SD Gangane . VIVA VOCE IN ANATOMY TuiRD Epition SD Gangane, us, Faris Professor and Head Genetic Division, Department of Anatomy Grant Medical College and Sir JJ Group of Hospitals Mumbai Ex-Professor and Head Department of Anatomy Rajarshi Chhatrapati Shahu Maharaj Govt. Medical College Kolhapur ELSEVIER ELSEVIER A division of Reed Elsevier India Private Limited Copyrighted material Viva Voce in Anatomy, 3/e SD Gangane ELSEVIER A division of Reed Elsevier India Private Limited Mosby, Saunders, Churchill Livingstone, Butterworth-Heinemann and Hanley & Belfus are the Health Science imprints of Elsevier: © 2011 Elsevier First Edition 1995 Second Edition 2006 Third Edition 2011 All rights reserved. No part of this publication may be reproduced, stored in a retrieval system, or transmitted in any form or by any means, electronic, mechanical, photocopying, recording, or otherwise without the prior written permission of the publisher. ISBN: 978-81-3 12-2706-0 Medical knowledge is constantly changing. As new information becomes available, changes in treatment, procedures, equipment and the use of drugs become necessary. The authors, editors, contributors and the publisher have, as far as it is possible, taken care to ensure that the information given in this text is accurate and up-to-date. However, readers are strongly advised to confirm that the information, especially with regard to drug dose/usag. complies with current legislation and standards of practice. Please consult full prescribing information before issuing prescriptions for any product mentioned in this publication. Published by Elsevier, a division of Reed Elsevier India Private Limited, Registered Office: 622, Indraprakash Building, 21 Barakhamba Road, New Delhi-110 001 Corporate Office: \4th Floor, Building No, 10B, DLF Cyber City, Phase Il, Gurgaon-122 002, Haryana, India. Managing Editor: Shabina Na Copy Editor: Neha Thapa Manager-Publishing Operations: Sunil Kumar Manager-Production: NC Pant im Typeset by Chitra Computers, New Delhi Printed and bounded at Sanat Printers, Kundli, Haryana. Contents Preface Part I: Chapter 1. Chapter 2. Chapter 3. Chapter 4. Chapter 5. Chapter 6. Chapter 7. Part Il: Chapter 8. Chapter 9. Part II: Chapter 10. Chapter 11. Part IV: Chapter 12. Chapter 13. Anatomy General Anatomy Superior Extremity Thorax Inferior Extremity Abdomen and Perineum Head and Neck Nervous System Histology General Histology Systemic Histology Embryology General Embryology Systemic Embryology Living Anatomy and Radiology Living Anatomy Radiological Anatomy and Imaging 1-212 3 19 44 66 103 150 192 213-272 215 239 273-322 275 292 323-366 325 342 Copyrighted material Copyrighted material image not available image not available image not available ay EE What do you understand by abduction and adduction? Qa Generally, abduction means moving away from the median plane while adduction means moving towards the median plane (Fig. 1.2). Midline Abduction ' Adduction Fig. 1.2. Abduction and adduction. What do you understand by the terms protraction and retraction? Protraction means moving anteriorly (forward) while retraction means moving posteriorly (backwards). What are dermal (membrane) bones? Qa They are the bones that develop in a membrane, e.g. bones of the vault of the skull. Match the bone with its function. Q Bone Function 1, Sesamoid bone (a) Offering resonance to voice 2. Pneumatic bone (8) Protection 3. Flat bone (c) Transmission of force 4, Long bone (d) Altering direction of muscle pull 5. Miniature long bone (2) Weight transmission Answer: 1 ~ (d),2 - (a), 3 - (b), 4 - (e), 5 - ()- Enumerate the bony sites where red marrow is found through- out life. The red marrow is found throughout life at following sites 1. Cancellous ends of long bones 2. Sternum Copyrighted material image not available image not available image not available a ER Name the sites where deep fascia is absent. a (i) In the face to permit movements of facial expression and mastication. (i) In the ventral abdominal wall to retain its expansile nature. (lif) Over subcutaneous bones. Q Enumerate the modifications of the deep fas The deep fascia is modified to form the following: 1. Intermuscular septa, epimysium, perimysium, endomysium in and around the muscles. 2. Around nerve and nerve fibres, epineurium, perineurium and endoneurium. 3. Sheaths around vessels, e.g. carotid sheath and axillary sheath. 4. In relation to joints, capsule, bursae, tendon sheaths, retinaculi around wrist and ankle joints. 5. Palmar and plantar aponeuroses. ‘ance? a What is a dermatome? What is its sign Adermatomeis an area of the skin supplied by one segmental nerve.Sensory loss in a dermatome reflects over the involvement of a corresponding segment of the spinal cord. What are Langer's lines? Qa These are cleavage lines. In the trunk they are arranged horizontally around the body wall. Incisions along these lines heal faster without much scar formation. Why is hyaline cartilage so called? Q Greek word hyalos means transparent stone. Since the matrix in this cartilage appears homogeneous, it looks transparentin fresh state, hence the name. What do you mean by ‘syndesmology’? Greek word syndesmos means ligament; syndesmology is thus the study of ligaments and related joints. Define joint. Q Joint is a junction of two or more bones or cartilages that allows movement. Who has more joints, a child or an adult? A child has more joints. As the growth proceeds, some of the bones fuse Copyrighted m image not available image not available image not available ery What is neuropathic joint? Denervation of joint renders it vulnerable to mechanical damage leading to swelling, excessive mobility and bone destruction. It occurs in tabes dorsalis, syringomyelia and leprosy. How is the word ‘muscle’ derived? Itis derived from a Latin word mus meaning mouse; because the tendons of some muscles look like the tail of a mouse hence the name muscle. What are slow and fast muscle fibres? The slow (type |) fibres are red in colour (due to more myoglobin), exhibit slow tonic contraction as in postural muscles. The fast (type II) fibres are paler (due to scanty myoglobin). They show fast phasic contraction and are easily fatigued. What is a motor unit? The motor unit, also called ‘myone’ consists of an alpha motor neuron along with the muscle fibres supplied by it. Match columns I and II for nomenclature of muscle. a T W 1. Triceps brachii (a) According to direction of fibres 2. Semitendinosus (b) According to number of heads 3. Stylohyoid (c) According to shape 4. Rectus abdominis (d) According to gross structure 5. Trapezius {e) According to attachments Answer: 1 - (b),2 - (d),3 - (e), 4 - (a), 5 - (o). Where do you get ‘en-grappe terminals’? These are specialized motor nerve endings that resemble a bunch of grapes. They are found in extraocular muscles and also in the endings of muscle spindles (Fig. 1.11). What is motor end plate? Q Itis cholinergic, specialized motor nerve ending. It is also called‘en plaque terminal’ (plaque = plate), They are found in skeletal muscles. What are the functions of vinculi longi? Q 1. To anchor the tendon to the phalanges. 2. To convey blood supply to the tendons. Copyrighted material image not available image not available image not available Er EE Enumerate the sites where sweat glands are absent. Q Sweat glands are absent in lips, glans penis and nail bed. What are ceruminous glands? These are apocrine sweat glands of the external acoustic meatus. They secrete ‘cerumen’ (ear wax). Name the sites where sebaceous glands are absent. Q They are absent in the skin of palms and soles. What is ichthyosis? Itis characterized by abnormally dry skin. The sebaceous glands are less in number and small; this markedly reduces the sebum secretion and leads to dry skin. What is the significance of transverse ridges on the nail? & During acute illness the toxins arrest the nail growth. After the illness the growth is resumed, this is marked by transverse ridge. Hence, transverse ridges on the nails indicate past illness (Fig. 1.13). Transverse ridge Lunule Fig. 113 Transverse ridges on the nail. Enumerate the sites where superficial fascia is devoid of fat. 4 (i) Eyelids, (ii) external ear, (iii) penis, (iv) scrotum and (v) flexion creases of digits. What is the average rate of growth of a nail? Qa Itis about 0.1 mm per day or 3 mm per month. It needs about 90-120 days for a complete nail to grow, ie. a period required for the treatment of fungal diseases involving nails. Copyrighted material image not available image not available image not available a Ee Which is more frequent, an anterior or a posterior dislocation of shoulder joint and why? Itis an anterior dislocation which is more frequently met with.|t is due to inherent weakness of the capsule of the shoulder joint anteriorly due to bursa under subscapularis muscle communicating with the joint cavity. How does subacromial bursitis clinically manifest? Q Itis usually due to inflamed and calcified tendon of supraspinatus. Clinically it presents with shoulder pain in first 15° abduction. However, beyond 15° the patient does not experience pain because then the inflamed bursa is away from the supraspinatus tendon. What is painful arc syndrom It is characterized by shoulder pain during 60° to 120° of abduction. It is due to chronic thickening of the supraspinatus tendon pressing against the coracoacromial arch. What are the branches of upper trunk of brachial plexus? & Suprascapular nerve and nerve to subclavius. What is the importance of the nerve to subclavius? Q At times it issues an accessory phrenic nerve (C, fibres) which joins the phrenic and supplies the diaphragm. Enumerate the structures lodged in the bicipital groove. Q (i) Tendon of the long head of biceps, (ii) its synovial sheath and (iii) ascending branch of the anterior circumflex humeral artery (Fig. 2.1). Synovial sheath Long head of biceps Ascending branch of anterior circumflex humeral artery Fig, 2.1 Contents of the bicipital groove. Name the structures piercing the clavipectoral fascia. Q () Thoracoacromial trunk, (i) lateral pectoral nerve, (ii) cephalic vein and (iv) lymphatics from the deep surface of the breast (Fig.2.2). Copyrighted material image not available image not available image not available a Ee 2. Radial half of the flexor digitorum profundus 3. Pronator quadratus (Fig. 2.7) Anterior interosseous nerve FOP Fig. 2.7 Distribution of anterior interosscous nerve (FPL, flexor pollicis longus; FDP, flexor digitorum profundus; PQ, pronator quadrates). In Erb's paralysis, what is the position of the upper limb? =& Waiter’s tip position, i.e.arm by the side of the body, elbow extended and forearm pronated (Fig. 2.8). Fig. 2.8 Erb’s paralysis: ‘the tip position’. What does Erb’s paralysis involve? Q It involves upper trunk of the brachial plexus, i.e. ventral rami of C, and fon In Klumpke’s paralysis what is the nerve injury? Q Lower trunk of the brachial plexus is injured. Medial cord of the brachial plexus is involved, i.e. ulnar nerve and medial root of median nerve are knocked down. Copyrighted material image not available image not available image not available Ey Sea What is crutch palsy? a Injury to the radial nerve being pressed by hard crutch. The result is wrist drop; since the extensors of the wrist are supplied by posterior interosseous, a branch of the radial nerve is paralysed. What is pronator syndrome? Itis entrapment neuropathy involving the median nerve. It presents with pain and tendemess in the proximal part of the forearm, weakness of muscles supplied by the median nerve and sensory impairment in the palm. What is cubital tunnel syndrome? Itis entrapment neuropathy of the ulnar nerve under the tendinous arch of origin of flexor carpi ulnaris between its humeral and ulnar attachments. Itpresents with flexor digitorum profundus involvement (the part supplied by ulnar nerve). What is student’s elbow? a It is also called miner's elbow. It is characterized by inflammation of subcutaneous olecranon bursa, with rounded, fluctuating and painful swelling little above the olecranon. What is tennis elbow? a Itis characterized by inflammation of tissues around lateral epicondyle of the humerus. |t is caused by spasm of radial collateral ligament or tearing of extensor carpi radialis brevis (ECRB) fibres or inflammation of bursa under ECRB. There is pain and tenderness in the lateral epicondyle. What is Golfer's elbow? It is characterized by pain and tenderness due to inflammation in the region of medial epicondyle of the humerus. It is due to the spasm of ulnar collateral ligament or partial tear of ulnar collateral ligament/ common flexor origin. What is the direction of the fibres of interosseous membranes and why is it so? Q The fibres are directed downwards and medially carrying line of force from the radius down to the ulna (Fig. 2.13). Copyrighted matorial image not available image not available image not available ey What is a claw hand? Q It results from combined lesion of the median and ulnar nerves, basic deformity being hyperextended metacarpophalangeal and flexed interphalangeal joints. ician’s nerve? Why is the ulnar nerve called mu: Ulnar nerve is called musician’s nerve because it controls fine movements of the fingers through its extensive motor distribution. What is space of Parona? Q Itis a quadrangular space located in front of the pronator quadratus in the lower part of forearm and behind long flexor tendons. Match the deformity with the nerve damage. Nerve damage Deformity 1. Ulnar nerve in hand (a) Wrist drop 2. Long thoracic nerve (b) Ape thumb deformity 3. Radial nerve in arm (©) Winging of scapula 4, Median nerve at wrist (d) Claw hand Answer: 1-(d), 2-(c), 3-(a), 4~-(b). What is Dupuytren’s contracture? Qa Aclinical entity resulting from a contracture involving especially the medial side of palmar aponeurosis. Out of the two terminal branches of the ulnar nerve which one can be rolled against the hook of hamate? Superficial terminal branch. Why is the lower end of the radius larger than that of the ulna? a Itis because the radius transmits line of force at the lower end while the ulna does not. What are the functions of interosseous membrane? a 1. Transmits force from the radius to the ulna. 2. Keeps these two bones (radius and ulna) together. 3. Provides extra surface for muscular attachment. 4. Isolates flexor from the extensors. Copyrighted material image not available image not available image not available Ey Ee Name the structures related to the back of clavicle. (i) Termination of the subclavian vein, (ii) commencement of the brachiocephalic vein, (iii) trunks of the brachial plexus, (iv) third part of the subclavian artery, (v) suprascapular vessels and (vi) lower end of the intemal jugular vein. What are the parts of coracoclavicular ligament? (i) Conoid part and (ii) trapezoid part. What are the peculiarities of clavicle? Q 1. The only long bone having two primary centres and intramembranous ossification. 2. Subcutaneous throughout its extent 3. Earliest to start (i.e. 5th week) and last to complete (i.e. 25 years) its ossification. 4. Lacks medullary cavity but contains bone marrow. 5. Sometimes pierced by subcutaneous nerves (supraclavicular). 6. The only long bone placed horizontally. Which is the common site of fracture of the clavicle? Q At the junction of medial 2/3 with the lateral 1/3. What is the effect of fracture involving clavicle medial to the attachment of coracoclavicular ligament? Line of weight/force transmission being interrupted and the shoulder drops down (Fig. 2.23). Fracture | Coracoclavicular ligament Cc. Fig. 2.23 Fracture of clavicle with interruption in the line of weight/force transmission. Copyrighted material image not available image not available image not available a EE What is meant by morphological neck of the humerus? Itis a junctional zone where upper epiphysis and diaphysis of the humerus join. It is represented by a line drawn horizontally at the lowest point of the articular surface of the humeral head. Which is the growing end of the humerus? Upper end (nutrient foramen gives the clue). What is related to the surgical neck of the humerus? 1. Posterior circumflex humeral artery. 2. Axillary (circumflex) nerve. What are the anatomical facts associated with the level of insertion of coracobrachialis? 1, Attachment of the deltoid on anterolateral surface. Q 2. Radial nerve piercing the lateral intramuscular septum goes to the anterior compartment. 3. Ulnar nerve piercing the medial septum goes to the extensor compartment. 4, Median nerve crosses over the brachial artery from lateral to medial sides (Fig. 2.28). Median nerve Uinar nerve Coracobrachialis Intermuscular Deltoid septum Radial nerve Humerus Fig. 2.28 Changes at coracobrachialis insertion. What structures are related to the spiral groove of the humerus? 1. Profunda brachii artery 2. Radial nerve Copyrighted material image not available image not available image not available Thorax Name the bony elements forming the thoracic inlet. (i Manubrium sterni, anteriorly, ii) first ribs, sideways, and (ili) Ist thoracic vertebra, behind. How is the subdivision of the mediastinum done? Qa It is divided by an imaginary plane passing through sternal angle and lower border of T, into superior and inferior mediastina. Inferior mediastinum is further subdivided into anterior (in front of the heart), middle (where the heart lies) and posterior (mediastinum which falls behind cardia) (Fig. 3.1). Superior mediastinum Sternal angle Be 30 |} — Inferior (mediastinum oO 5 CO Fig. 3.1 Thoracic cavity (1, 2 and 3 are anterior, middle and posterior mediastinum, respectively). Copyrighted material image not available image not available image not available ay Arch of aorta Ductus arteriosus Left pulmonary artery Accessory phrenic nerve Phrenic nerve Nerve to subclavius Fig. 3.6 Accessory phrenic nerve. What is endothoracic fascia? It is a thin areolar layer between the pleura and the thoracic wall. It corresponds to fascia transversalis of the abdominal wall. Name the parts of transversus thoracis muscle. (i) Subcostalis, (ji) intercostalis intimi and (iii) sternocostalis. What is pulmonary ligament and what is its function? It is a double-layered pleura that hangs below the root of the lung and provides potential space for dilatation of pulmonary vessels. What is the difference between lung root and hilum of lung? Q Hilum of lung is an area (two dimensional) through which various structures either enter or leave the lung. Lung root is a structure with three dimensions constituted by all the structures entering or leaving the lung (Fig. 3.7). Copyrighted material image not available image not available image not available ay Ee In posteroanterior (PA) view of the chest, what constitutes the hilar shadow and how does it look like? In X-ray chest PA view, the hilar shadow looks like @ paint brush. |t consists of the following structures: (i) pulmonary vessels, (i) bronchial shadow, (ii) lymph node (hilar) and (jv) interstitial tissue. What is the blood supply of lung? = Itis furnished by bronchial arteries,as visceral branches of the descending thoracic aorta (and not by pulmonary arteries], two on the left side and ‘one on the right side, arise from either third posterior intercostal or upper left bronchial artery. Name two diseases which cross the barrier of bronchopulmo- nary segments. (i) Pulmonary tuberculosis and (ii) carcinoma of the lung. What are the vertebral levels of commencement and termina- tion of trachea? It begins at C, and terminates at T,. Why do tracheal rings present deficiency posteriorly? Q Posterior deficiency in the tracheal cartilages is occupied by trachealis muscle which provides soft cushion that allows for expansion of the oesophagus during swallowing and also tracheal expansion during coughing (Fig. 3.11). Oesophagus ‘Trachealis Trachea Fig. 3.11 Trachealis muscle acting as soft cushion for the oesophagus. What type of cartilage forms tracheal rings? Tracheal rings consist of hyaline cartilage. They cannot have elastic cartilage, because otherwise the trachea will form a collapsible tube and being an air passage it cannot afford this. Copyrighted material image not available image not available image not available Sea What is angina pectoris? Q Itrefers to an entity resulting from myocardial ischaemia and is associated with severe pain on exertion. Pain is referred to the neck and the inner aspect of the left arm. However, in diabetics myocardial ischaemia may not be associated with pain (owing to diabetic neuropathy). It is called silent myocardial ischaemia. Why and where does referred pain from the heart appear? & Pain sensation from the heart is carried by visceral afferent fibres (sympathetic), neurons of which are located at T,-T,. Pain impulses intermingle with somatic afferent neurons in such a way that somatic painis also simultaneously experienced. Cutaneous area of the precordium is supplied by T,-T, segments and along the inner aspect of the arm it is T, segment, that of the forearm it is T, segment, therefore cardiac pain is referred to the pericardium as well along the inner aspect of the arm and forearm, i.e.T,-T, segments. What is triple vessel disease? Q An average narrowing of the following three major arteries supplying the heart is called triple vessel disease. 1. Anterior interventricular branch of left coronary artery about 40-50% 2. Right coronary artery (RCA) (30-40%) 3. Left circumflex artery (15-20%) What is PTCA? Q PTCA stands for percutaneous transluminal coronary angioplasty. Through an artery in the arm or leg, a balloon catheter is guided into coronary artery, reaching the obstruction, the balloon is inflated. This compresses the obstructing plague against the vessel wall; however, restenosis may occur in about 6 months. Better alternative being the insertion of ‘stent’ (stainless steel device), which offers better results. What is aortic knuckle? Qa Itis the prominence caused by the arch of aorta as it projects beyond the left margin of the sternum in X-ray chest posteroanterior (PA) view. Aneurysm of the aorta leads to what kind of a symptom complex? 1. Hoarseness of voice—Because of the involvement of the left recurrent laryngeal nerve. 2. Dry cough—Pressure on the trachea. image not available image not available image not available ay Ee 3. Oblique musculature of the stomach forming an inverted U-shaped loop. 4. Fold of the mucous membrane projecting at the gastro-oesophageal junction (Fig. 3.19). Angle between oesophagus and stomach Zo Oblique muscle fibres Fig. 3.19 Factors operating, as cardiac sphincter. Where does the thoracic duct commence and terminate? a It commences at T,, as a continuation of cisterna chyli and terminates at jugulo-subclavian angle. When is the thoracic duct likely to be injured? During sympathectomy. Why is the azygos vein so called? a Because it has no companion vein (unpaired one). How is azygos vein formed? By union of the right ascending lumbar vein and the subcostal vein, or it may arise from the back of the IVC at the level of renal vein. Name the structures accompanying the azygos vein in its passage through the diaphragm. Qa Itis accompanied by aorta and thoracic duct. Alll the three structures pass through the aortic orifice in the diaphragm at T,, (Fig. 3.20). Which embryonic vessel gives rise to the terminal part of azygos vein? Itis derived from the right posterior cardinal vein. Copyrighted material image not available image not available image not available a What is a cervical rib? Q Itis attached to C, (7th cervical vertebra) and occurs with frequency of 0.5% in population. It causes pressure on trunks of the brachial plexus and adjacent vessels also, leading to neurovascular symptoms. Why coarctation of aorta is associated with notching of the ribs? Q In coarctation of aorta, the posterior intercostal arteries are enlarged in order to provide collateral circulation. These enlarged arteries exert pressure and cause notching on the ribs. What is stove in chest/flail chest? In severe compression injuries of the chest, the vertebrosternal ribs are fractured at both ends (anterior as well as posterior). Therefore, a part of the chest (thoracic cage) is almost free. It shows paradoxical respiratory movements, ie. it is sucked in on inspiration and blows out on expiration. This is called flail chest or stove in chest. What is pigeon chest? Congenital defect of the chest in which the sternum projects anteriorly. What is rachitic rosary in rickets? Q In rickets, the sternal ends of second to eighth ribs look like beads (enlarged at costochondral junctions). This is called rachitic rosary. What is thoracic inlet syndrome? Itis characterized by pressure on the subclavian artery and the first thoracic nerve crossing the first rib. The pressure may be exerted by the cervical rib or by variation in insertion of the scalenus anterior muscle. It presents with vascular, neural or both symptoms. What are the functions of intervertebral disc? a The functions of intervertebral disc are as follows: 1. They give shape to the vertebral column (responsible for the curvature/ flexures). 2. They act as shock absorbers/buffers. 3. They allow movement. This is duly attested by the fact that intervertebral discs in cervical and lumbar regions are thicker. What is kyphosis? Q Itis a deformity involving the spine in sagittal plane. It is also called round Copyrighted material image not available image not available image not available a ee Where do the superficial veins accompanying the three super- ficial branches of the femoral artery terminate? In the long saphenous vein and not in the femoral vein. Where do you look for inflammatory focus in the enlargement of inguinal lymph nodes apart from the lower limb? (i) Terminal part of the anal canal, (ji) external genitalia and (iii) fundus of the uterus along the round ligament of the uterus. Why is long saphenous vein so called? Q Because it is a long and easily seen vein (saphenous = clear, prominent). Where does long saphenous vein start and terminate? It commences at the medial end of the dorsal venous arch after being joined by a vein from the medial side of the great toe. It terminates in the femoral vein after passing through the saphenous opening. What is the purpose of valves in the long saphenous vein? There are about 18-20 valves along the course of the vein. They serve to support the column of blood to be propelled against gravity. Name the factors responsible for antigravity flow of blood in the long saphenous vein. Q (i) Valves in the vein, (ii) calf muscles serving as the peripheral heart and (iii) negative intrathoracic pressure. What is the anatomical basis of varicosity of veins in the lower limb? 1. Damage to valves in the long saphenous vein near its termination. 2. Damage to perforators connecting the superficial and deep veins. In both the events, Brodie-Trendelenburg test helps to locate the site of defect. What are the causes of varicosity of veins? Qa 1, Inherent weakness of valves 2. Thrombophlebitis, i.e. inflammation involving veins 3. Obesity 4. Pregnancy 5. Long standing Copyrighted material image not available image not available image not available i Straight head Reflected head Rectus femoris Fig. 4.8 Attachment of the rectus femoris to the hip bone. What is the function of articularis genu? Q Itprevents synovial membrane from getting caught in between the two bones of knee joint during forcible extension of the knee (Fig. 4.9). Atticularis genu ‘Synovial fold — bS Fig. 4.9 Articularis genu pulling the synovial membrane up. Enumerate the parts of quadriceps femoris. Q (i) Rectus femoris, (ii) vastus medialis, (ii) vastus intermedius and (iv) vastus lateralis. Where is quadriceps femoris inserted? To tibial tuberosity through the ligamentum patellae (see Fig. 1.3). What is ligamentum patellae? a Itis a morphologically degenerated part of the quadriceps femoris. What is the function of tensor fasciae latae? By virtue of its attachment to the iliotibial tract, it tenses the iliotibial tract. The latter serves function of balancing the pelvis over femoral heads. Copyrighted material image not available image not available image not available image not available image not available image not available image not available image not available image not available image not available image not available Wace cont Out of the two semilunar cartilages which is more vulnerable to injury and why? Q Medial semilunar cartilage (meniscus) is more vulnerable and gets injured in forcible extension of the knee accompanied by lateral rotation, as in football game. To the medial meniscus tibial collateral ligament is attached, which does not allow its prompt withdrawal as is possible for lateral meniscus with the help of popliteus that pulls it out and prevents injury to it (Fig. 4.23). Medial meniscus Popliteus Tibial collateral ligament Fig. 4.23 Two menisci of the knee joint. What is varus deformity? In this, lower end of the distal bone forming the joint is shifted medially, e.g. coxa vara. Herein the lower end of femur is deviated medially, i.e. the neck-shaft angle is reduced. From where can the effusion of knee be tapped? On medial or lateral sides of the patella. What is the function of cruciate ligaments? a 1. To stabilize the joint by preventing dislocation in anteroposterior direction (Fig. 4.24). 2. Anterior cruciate ligament, getting taut serves as an axis for rotation at knee joint during terminal (30°) stages of extension. What is genu valgum? Q It is also called knock-knee, where angulation between femur and tibia is increased and is opened laterally (Fig. 4.25). Copyrighted material image not available image not available image not available i Which muscles are crossed by the tendon of flexor digitorum longus? Tibialis posterior in the leg and flexor hallucis longus in the sole. Name the muscles that originate from the tendon of flexor digitorum longus. Also name the muscle that is inserted into it. Q Lumbricals arise from the digital slips of tendon while flexor digitorum accessorius muscle inserts on the proximal side of the tendon of flexor digitorum longus (Fig. 4.28). Ist lumbrical IVth lumbrical Flexor digitorum longus Flexor digitorum accessorius Fig. 4.28 Attachments to the tendon of flexor digitorum longus. What are the functions of calf muscles? 1. Plantar flexion of ankle, raising the heel off the ground in propulsive phase of walking. 2. Flexion of knee. 3. Aids the antigravity flow of the blood in veins (peripheral heart). Name the nerves supplying the first web of the foot. Q 1. Dorsally—By medial terminal branch of the anterior tibial nerve. 2. On plantar side—By digital branch of the medial plantar nerve. Name the muscles attached to the big toe. (i) Extensor hallucis longus and brevis, (ji) flexor hallucis longus and brevis, (iii) flexor digitorum brevis, (iv) abductor hallucis brevis and (v) adductor hallucis. Name the artery that replaces dorsalis pedis when the latter is absent. Perforating branch of the peroneal artery.|tcomes to extensorcompartment Copyrighted material image not available image not available image not available ay Ee Horizontal --.2.- i bisa plane Coronal plane Fig. 4.30 Anatomical position of the hip bone. Acetabulum ——— Ischial spine Fibrofatty tissue Fig, 4.31 Part of ischial tuberosity sitting posture. Why is femoral shaft convex anteriorly? Q Because the line of gravity passes behind the hip joint and in front of the knee joint (Fig. 4.32). Why are femora more obliquely placed in females? Because of the broader pelvis separating femoral heads. What is linea aspera and what is its significance? Itis a rough (aspera) thickened strip on the back of femoral shaft reinforcing it and compensating for weakness created by forward convexity of the shaft. Copyrighted material image not available image not available vero nce a the gluteus maximus reaches bone, secondly medial line is rounded off by femoral vessels crossing it (Fig. 4.35). Lateral intermuscular septum Femoral vessels Adductor tubercle Fig. 4.35 Posterior view of the lower end of the femor showing supracondylar lines and the related structures. Q In the anterior part of the popliteal groove, while the posterior part of the groove lodges the tendon of popliteus during full flexion of the knee (Fig. 4.36). Where is popliteus attached to the fem Lateral epicondyle For attachment For the tendon of popliteus Fig. 4.36 A view of popliteal groove. What is the medicolegal importance of the ossification of lower end of the femur? Qa Centre of ossification for this end appearsin the 9th month of intrauterine life and therefore the presence of this centre in a newborn found dead is a proof that the child was viable. What is the importance of adductor tubercle? a The importance of adductor tubercle is that the lower epiphyseal plate passes through it. Therefore, before any surgical intervention in this area, one has to essentially note that it is the growing end of the bone and hence disturbance to this epiphyseal cartilage might lead to permanent shortening of the limb (Fig. 4.37). Copyrighted material Viva Voce in Anatomy Synovial fold Adductor Epiohyseal tubercle plate Fig. 4.37 The lower epiphyseal plate passes through the adductor tubercle. What is the function of patella? Imagine patella as a stone put under the quadriceps going to tibia, this essentially makes the quadriceps to take up longer course, meaning that it adds to the functional length and hence the efficacy of contraction (Fig. 4.38). A B Fig. 4.38 Patella adding to the efficiency of quadriceps femoris: (A) lon- ger course and (B) shorter course. What is bipartite patella? More often it is the superolateral angle of this bone that remains as a separate bone (developing from separate centre). It may be mistaken for a fracture of the patella. What prevents the natural tendency of dislocation of the patella in lateral direction? Q 1. Forward projection of the lateral condyle of the femur Copyrighted material Monee 2. Medial pull exerted by fibres of the vastus medialis. 3. Medial patellar retinaculum is thicker than lateral (Fig. 4.39). 4 ae Lateral condyle Fig. 4.39 Factors preventing lateral dislocation of the patella (P, patella; F, femoral condyles). Veastus medialis Mention the sequence of attachments of menisci and cruciate ligaments on superior surface of the tibia (intercondylar area). From before backwards, these are (i) anterior horn of medial meniscus, (ii) anterior cruciate ligament, (iii) anterior horn of lateral meniscus, (iv) posterior horn of lateral meniscus, (v) posterior horn of medial meniscus and (vi) posterior cruciate ligament (Fig. 4.40). Anterior horn of medial meniscus Anterior cruciate ligament Anterior and posterior horn of lateral Posterior horn of meniscus medial meniscus Posterior cruciate ligament Fig. 4.40 Attachments on the superior aspect of tibia. Why, of the two bones of leg, tibia is stronger than fibula? Because tibia transmits weight and fibula does not. What are the joints formed by the tibia with other bones? 1. With femur—Knee joint 2. With fibula—Superior tibiofibular (plane synovial) joint 3. With middle—Tibiofibular (fibrous) joint Copyrighted material Ey SE 4. With inferior—Tibiofibular (syndesmosis) joint 5. With talus—Talocrural (ankle hinge) joint Name the muscles attached to the medial surface of tibia. % (i) Sartorius (extensor compartment), (ii) gracilis (medial compartment) and (iii) semitendinosus (flexor compartment) (Fig. 4.41]. Sartorius i Gracilis —— Semitendinosus Fig. 4.41 Attachments to the medial surface of tibia. Note all these mus- cles are attached to the upper part only. The remaining whole medial surface is subcutaneous (F, fibula; T, tibia). What grooves the back of medial malleolus? Tendon of tibialis posterior. Of the two malleoli which is shorte Medial malleolus. Name the fascia to which the soleal line offers attachment. % (i) Popliteal fascia, (ii) soleus muscle and its covering fascia, (iii) fascia covering deep muscle of the leg and (iv) medial end of the tendinous arch of soleus (Fig. 4.42). Popliteal fascia ‘Tendinous arch Soleus ~~ Fascia over deep muscles of leg Fig. 4.42 Structures attached to the soleal line of tibia. Copyrighted material Fret oncreent a Vertical line on the back of tibia isolates attachments of which muscles? Tibialis posterior (laterally) from flexor digitorum longus (medially). What is the function of fibula? Q 1, Provides resilient lateral malleolus in tibiofibular mortise to accommodate the broader anterior part of talus during dorsiflexion of ankle. 2. Offers area for muscular and ligamentous attachments. 3. Serves as a pulley for the tendons of peroneus longus and peroneus brevis (Fig. 4.43). Talus Fig. 4.43 Resilient lateral malleolus (F, fibula; T, tibia). Name the structure related to the medial crest of fibula. Peroneal artery. How and why does the fibula disobey the law of ossification? % Centre for its upper end appears lateand also fuses late.Possible explanation being that the tractional force of biceps tendon does not allow fusion between it and the diaphysis. However, after tibial end has completed its ossification the upper end of fibula also fuses (Fig. 4.44). Fracture involving the neck of fibula leads to foot drop, which is the structure sacrificed? Common peroneal nerve, knocking down extensors (dorsi-flexors) of the ankle joint. What is tarsal tunnel syndrome? Paraesthesia with pain along the sole of foot owing to compression of the tibial nerve under the flexor retinaculum of ankle is called tarsal tunnel syndrome. Copyrighted material Biceps femoris EPphvssy Tibia Fibula Biosiis: Fig. 4.44 Fibula disobeys the law of ossification. What is Pott’s fracture? Qa With the foot caught in a hole, medial rotation of the limb causes (i) spiral fracture of the lateral malleolus, (i/) as the foot is everted, the avulsion of the medial malleolus follows.|n this condition, an attempt to free the foot causes (ii/) shearing off of the lower end of tibia. These three are referred to as the first, second and third degrees of Pott’s fracture. What is Dupuytren’s fracture? Extreme abduction at foot causes rupture of the interosseous ligament, membrane and fracture involving the lower end of fibula (3 inch from the tip of lateral malleolus). This is called Dupuytren’s fracture. Why is the position of foot inverted in young children? Because of smaller angle (130°-140°) between the long axis of neck and the body of the talus. What is congenital talipes equinovarus? Q It is commonly called club foot. In this, ankle is planter flexed, foot is invented at subtalar joints with medial border of the foot raised and forefoot is adducted. What is calcaneal spur? It is usually seen in a fatty, fertile female in her forties with flat foot. Increased body weight causes tear in plantar fascia near its attachment to the medial tubercle of calcaneus. Repeated tears with the haemorrhage in the periosteum lead to calcification and ossification forming bony spicule, the calcaneal spur. Copyrig nted matarial What is the cause of pain in a person with calcaneal spur? & Pain is due to reactive fibrosis involving periosteum. At which other sites can calcaneal spur occur apart from the medial tubercle? Calcaneal spur can be on the plantar aspect of bone at the attachment of short plantar ligament or on the posterior surface at the attachment of tendo calcaneus. What is os trigonum? Q Itis the tubercle of the talus that exists independently (Fig. 4.45). Os trigonum Fig. 4.45 Os trigonum (FHL, flexor hallucis longus). What occupies the groove between medial and posterior tu- bercles of the talus? Tendon of flexor hallucis longus (Fig. 4.46). LI Posterior 4} tubercle . Medial tubercle FHL Fig. 4.46 Flexor hallucis longus (FHL) tendon passing between the tu- bercles of talus. What is March fracture? Qa Commonly seen in soldiers and nurses with long standing jobs. Itis ¢ fatigue fracture which involves distal third of the second or third metatarsal bone. Copyrighted material What is hallux valgus? There is lateral deviation of the great toe. it may be due to wearing of pointed shoes or in football players kicking with forefoot. What is Hammer toe? Q Itis characterized by displacement of the second toe with acute flexion at the proximal interphalangeal joint and extension at the distal interphalangeal joint. Thus presenting like a hammer. The deformity is associated with pes cavus and is secondary to hallux valgus. What is Morton's metatarsalgia? Pressure on the lateral plantar nerve by the head of fourth metatarsal leading to intermittent pain in forefoot between third and fourth metatarsals is Morton's metatarsalgia. Which bony points are considered for measuring the length of lower limb? The length is measured from the anterior superior iliac spine to adductor tubercle and from there to the tip of medial malleolus. Copyrighted material image not available What is a transpyloric plane? Aplane passing through the pylorus midway between suprasternal notch and pubic symphysis. It passes through the tips of 9th costal cartilages and corresponds to L, vertebra (Fig. 5.1). Enumerate the structures lying at the transpyloric plane. & (i) Pylorus of the stomach, (ii) fundus of the gallbladder, (iii) neck of the pancreas, (iv) formation of the portal vein, (v) origin of the superior mesenteric artery and (vi) hilum of the right kidney. What are linea gravidarum? These are linear white scars, seen in the lower part of the ventral abdominal wall in women who have borne children. What is Cullen’s sign? It is an appearance of faint, irregular haemorrhagic patches on the skin around umbilicus. Itis observed in acute pancreatitis 1 or 2 days after the onset of anorexia and severe abdominal pain. What is grey Turner’ Observed in acute haemorrhagic pancreatitis. It is bruising of the skin of loin. sign? Normally at what level does the umbilicus lie? Is it a good surface landmark? It usually lies at the level of disc between L, and L,. The umbilicus is not a reliable landmark since its level is variable. If intestinal contents (faecal matter) make their way at the umbilicus, what is the embryological defect? Persistence of the vitellointestinal duct. What i A condition in which enlarged veins are seen radiating away from the umbilicus, viz. in portal hypertension (Fig. 5.2). put medusae? a Which nerve supplies the skin of anterior abdominal wall at the level of umbilicus? a Tenth thoracic nerve (T,,). Referred pain of appendicitis is experienced at the umbilicus. Copyrighted material Abdomen and Pe Paraumbilical vein Umbilicus Fig. 5.2 Caput medusae. How is the disposition of superficial fascia in the lower part of ventral abdominal wall? It is split into a superficial fatty layer (Camper's fascia) and a deeper membranous layer (Scarpa’s fascia). What is the peculiarity of the lateral cutaneous branch of the subcostal nerve? It does not divide into anterior and posterior cutaneous branches, unlike all other intercostals. Name the nerve crossing the tubercle of iliac crest. Lateral cutaneous branch of the iliohypogastric nerve. What is the significance of the lumbar triangle of Petit? Itis @ small triangular space bounded by the external oblique latissimus dorsi and iliac crest. Herniation may occur through the potentially weak spot. Name the superficial branches of the femoral artery. Q (i Superficial external pudendal, i) superficial epigastric and (iii) superficial circumflex iliac arteries (Fig. 5.3). Why do veins on the ventral abdominal wall become promi- nent in vena caval obstruction? Ventral abdominal wall above the umbilicus drains along lateral thoracic and internal mammary veins to the superior vena cava while below the umbilicus it is via the femoral vein to the inferior vena cava (IVC), thus there is communication between the two vena caval channels; therefore if one is blocked the blood goes via these channels to the other, making intervening veins prominent. Copyrighted material Superficial circumflex iliac Superficial epigastric Superficial —7 extemal pudendal 4] ps Femoral vessels Long —| saphenous vein Fig. 5.3 Superficial branches of the femoral artery. Which group/groups of lymph nodes are involved in carci- noma of the umbilicus? Q Both right and left axillary and right and left inguinal groups of lymph nodes (Fig. 5.4). Axillary nodes Umbilicus Inguinal nodes Fig, 5.4 Spread of carcinoma of the umbilicus. Name the muscular constituents of the ventral abdominal wall. (i) External oblique, (ii) internal oblique, (iii) transversus abdominis, (iv) rectus abdominis and (v) pyramidalis. How are the fibres of external and internal oblique muscles oriented? Q They are oriented at right angles to each other, thereby offering more strength to the abdominal wall to support viscera and to raise intra- abdominal pressure during forceful physiological acts (Fig. 5.5). Copyrighted material bdomen and I External oblique Internal oblique Fig. 5.5 Direction of the fibres of oblique muscles of the abdomen. Name the muscles interdigitating with external oblique at its origin. (i) Serratus anterior and (ii) latissimus dorsi. What is inguinal (also called Poupart’s ligament) ligament? Itis enrolled aponeurosis of the extemal oblique muscle. Why is inguinal ligament curved downwards? By virtue of an attachment of the deep fascia of the thigh below, whose downward pull makes it run in a curved manner (Fig. 5.6). Poupatt's ligament Pubic symphysis +} Deep fascia of thigh Fig. 5.6 Deep fascia of the thigh exerting pull on the inguinal ligament. What are the parts of inguinal ligament? Q (i Lacunar ligament/Gimbernat’s ligament, (i) reflected part and (ii) pectineal ligament (ligament of Cooper) (Fig. 5.7). Copyrighted material Femoral artery Reflected Femoral V part Ligament of Cooper / Femoral canal Lacunar ligament (Gimbernat’s ligament) Fig. 5.7 Parts of the inguinal ligament. What is McBurney’s incision? a Itis a muscle-splitting incision used for appendicectomy or colostomy. It is advantageous because it leaves no postoperative weakness; however, it provides limited exposure. What is cremaster muscle? a Itis considered as a part of the internal oblique muscle springing from middle of the inguinal ligament, forming series of loops at the superficial inguinal ring that envelope the testis and help in its elevation (Fig. 5.8). Pubic tubercle inguinal ligament Cremasteric loops Fig, 5.8 Attachments of the cremaster muscle. What is external spermatic fascia? It is a connective tissue envelope derived from the external oblique aponeurosis around constituents of the spermatic cord. Copyrighted material image not available Skeletal Muscular +—— Aponeurotic Umbilicus }—— Arcuate line Areolar Fig. 5.9 Posterior wall of the rectus sheath. Which two arterial trunks anastomose through superior and inferior epigastric arteries? Subclavian and external iliac arteries. What are the functions of ventral abdominal wall muscles? & . Supporting the abdominal viscera. 2. Regulating intra-abdominal pressure, raising it in all forceful physiological acts such as coughing, sneezing, forced micturition, defaecation, respiration, parturition, etc. 3. Movements of the vertebral column such as flexion, rotation and lateral bending. What lies superficial and deep to the fascia transversalis? Superficial to it is transversus abdominis and deep to it is extraperitoneal fat. Name the deficiency in lower part of the fascia transversalis. © Deep inguinal ring. What is the fate of fascia transversalis in the thigh? In the thigh it forms the anterior wall of the femoral sheath. What is the contribution of fascia transversalis to the sper- matic cord? It contributes a tubular sheath called internal spermatic fascia (Fig. 5.10). Copyrighted material image not available Inferior epigastric artery Rectus abdominis Deep inguinal ring Hesselbach's triangle Inguinal ligament Fig. 5.11 Boundaries of Hesselbach’s triangle. What is hernia? Qa It is the protrusion of a viscus or part of a viscus through an abnormal ‘opening in the walls of its containing cavity. Mention muscular mechanisms protecting the inguinal canal. . Obliquity of the canal, which increases by contraction of abdominal muscles. 2. Shutter mechanism/sphincter mechanism by internal oblique muscle. 3. Lytle demonstrated contraction of fibres on the deep surface of transversus abdominis pulling deep ring under lower edge of the transversus abdominis. 4. Ball-valve mechanism involving cremaster contraction, elevation of the testis, plugging the weak spot (external inguinal ring) (Fig. 5.12). EOE Shutter mechanism Sphincter mechanism Fibres of transversus abdominis internal oblique pulling the deep ring up a o Ball-valve mechanism, Fig. 5.12 Muscular mechanisms protecting the inguinal canal. Copyrighted material image not available Left suprarenal vein ve — ) Yr Left renal vein Right testicular vein J— Left testicular vein Fig. 5.13 Direction of emptying of testicular veins. Ostia of left testicular and left suprarenal veins lie opposite to each other. Aorta Superior mesenteric artery Left renal vein. Third part of duodenum Fig. 5.14 Note the position of left renal vein like a nut in a nutcracker which is formed by the aorta and the superior mesenteric artery. Where do lymphatics from the testis terminate? a In pre- and para-aortic group of lymph nodes. How to identify the side of the testis in a cadaveri By putting the organ in an anatomical position, with the sinus of epididymis opening laterally. specimen? What is aditus to lesser sac? It is also called epiploic foramen or foramen of Winslow. It is the communication between greater and lesser sac. What lies in the right free border of lesser omentum? a Portal vein (posteriorly) and bile duct and hepatic artery (anteriorly). The duct is right (dexter) to the hepatic artery. Enumerate the structures limiting the epiploic foramen. Q 1. Anteriorly by right free margin of the lesser omentum. 2. Posteriorly by the inferior vena cava. Copyrighted material ia 3. Superiorly by the caudate process of the caudate lobe of liver. 4. Inferiorly by the first part of duodenum (Fig. 5.15). bdomen and P: Wy 77 Cauidete process vc Epiploic foramen First part of duodenum Lesser omentum Fig. 5.15 Boundaries of the epiploic foramen. What are the functions of peritoneum? (i) Formation of the covering for the viscera, [ii) keeping the viscera in normal position, (iii) blood vessels pass along the peritoneal folds, (iv) prevention of adhesions in normal state, (v) absorption of the fluid from the peritoneal cavity and (vi) protection by restricting infection by adhesions and fixed macrophages. What is the capacity of the stomach? Q About 1500 ml (1.5 L)in normal adult. What are the parts of the stomach? a Part of the stomach above the level of cardiac orifice is called fundus of the stomach. Vertical portion is the body and horizontal parts are the pyloric antrum and the tubular pyloric canal (Fig. 5.16). Fundus Pyloric canal Fig. 5.16 Parts of the stomach. Copyrighted material What is gastric canal? Q Part of the stomach cavity along lesser curvature is isolated by oblique musculature of the stomach wall to form gastric canal (canal of Lewis) (Fig. 5.17). Gastric canal Oblique fibres ig. 5.17 Gastric canal or canal of Lewis. In a fresh specimen, what demarcates the pylorus from the duodenum? Prepyloric vein of Mayo which lies in front of the gastroduodenal junction. It serves as an important guide to a surgeon. In a cadaveric specimen, how are pyloric and cardiac ends of the stomach distinguished or identified? Pyloric end is thick because of the pyloric sphincterand its anterior surface shows biliary (green) staining. Enumerate the constituents of the stomach bed. a (i) Lesser sac, (i) body of the pancreas with splenic artery, (iii) left kidney, (iv) left suprarenal, (v) spleen, (vi) diaphragm, and (vi/) transverse colon and mesocolon (Fig. 5.18). Stomach Diaphragm Left suprarenal — Spleen Left kidney — 47) | Coeliac trunk -Q Ae Splenic artery Left colic flexure Transverse mesocolon Transverse colon Fig. 5.18 Stomach bed. Copyrighted material What is the nerve supply of the stomach? 1. Vagal supply—(i) Secretomotor to glands and (ii) motor to the musculature of stomach wall, and inhibits the pyloric sphincter. 2. Sympathetic (T,_, segments) supply—lt stimulates the sphincter. What are the functions of the stomach? a (i) Digestion of food, (ii) production of intrinsic factor for absorption of vitamin B,,, (iii) absorption of some substances, alcohol, drugs, etc., (iv) secretion of HCI which is bactericidal, (v) secretion of the hormone gastrin which regulates pepsin and HCl secretion and (vi) reservoir for food. What are the different shapes of stomach? & 1. 4-shaped, normal/typical stomach 2. Hourglass stomach 3. Steer horn stomach What is the weight of the spleen? a 7 ounces, ie. 80-300 g (1 oz = 28.3 g). What inference would you draw if the spleen is palpable? Normally it is not palpable. But if it is palpable it indicates enlargement of the spleen (splenomegaly). Enumerate the causes of splenomegaly. Q Splenomegaly (enlarged spleen) is caused by (i) chronic malaria, (i) portal hypertension, (iii) haemolytic anaemia, (iv) chronic myeloid leukaemia, (v) myoproliferative disorders, (vi) typhoid, (vii) kala azar and (viii) Niemann— Pick disease, etc. On enlargement of the spleen, in which direction does it grow? & Towards the right iliac fossa (Fig. 5.19). How would you distinguish between an enlargement of the spleen and that of the left kidney? Qa In splenic enlargement (i) lump presents notches, (ii) one cannot insinuate fingers under the left costal margin, (iii) it moves with respiration, (iv) there is a dull percussion note in splenic lump, while it is not so in renal lump because of overlying intestinal coils, and (v) renal lump is palpable bimanually. Copyrighted material Spleen Right iliac fossa Fig. 5.19 Direction of splenic enlargement. What is the significance of notched superior border of the spleen? Q It signifies lobulated origin of the spleen. Why is the splenic artery tortuous? Q So thatit can permit displacement of the spleen caused by a full stomach and by movements of the diaphragm without putting strain on the arterial trunk. Why is lienorenal ligament considered important? Q Because it contains within its two layers, splenic vessels and the tail of pancreas (Fig. 5.20). Splenic vessels Tail of pancreas Pancreas Fig. 5.20 Contents of the lienorenal ligament. What are spleniculi? These are small nodules of the splenic tissue found close to the main splenic mass. Copyrighted material CE What is the relationship of the weight of liver to the body weight? In an infantitis 1/18th of the body weight, however in an adultitis 1/36th of the body weight. How does a voluminous organ like liver maintain its position? Q 1. Liver is nailed down by hepatic veins to the inferior vena cava. 2. In addition, it is duly supported by intra-abdominal pressure and adjacent viscera. How much of the hepatic tissue is destroyed before liver func- tion tests give clear positive indication? About 80%. What is ligamentum teres hepatis and what does it represent? Ligamentum teres hepatis extends from the umbilicus to the left end of porta hepatis. It represents the obliterated left umbilical vein. What does ligamentum venosum represent? Q It represents the obliterated ductus venosus, connecting the left branch of portal to the left hepatic or to the inferior vena cava. What is the purpose of ductus venosus in prenatal life? Q Purpose of ductus venosus is to divert better oxygenated blood coming along the left umbilical vein via the left branch of portal vein to the left hepatic and then to the inferior vena cava without being poured in the hepatic sinusoids. From the inferior vena cava it goes across the foramen ovale to the left atrium, to the left ventricle then to aorta and by branches of the arch of aorta to vital organs in the head and neck. What are the bare areas of the liver? a . Triangular bare area between the two layers of the coronary ligament (base being formed by a groove for the inferior vena cava). . Fossa for the gallbladder. . Porta hepatis. Between two layers of the peritoneum forming falciform ligament, triangular ligament and fissure for ligamentum venosum (Fig. 5.21). RWN Copyrighted material — ve Left triangular ligament Bare area Fissure of ligamentum venosum Round ligamént Porta hepatis Fossa for gallbladder Fig. 5.21 Bare areas of the liver. Why is the left lobe of the liver smaller than the right? a Because ductus venosus diverts blood directly to the inferior vena cava, thus depriving the developing left lobe of portal blood. What is the blood supply to the liver? Itis supplied by the hepatic artery and portal vein. What is the cause of portal hypertension in cirrhosis of the liver? Q In cirrhosis of the liver, there is fibrous tissue replacing hepatic sinusoids which normally mediate pressure gradient between high pressure in arterial and portal radicles. Once hepatic sinusoids are destroyed, the high arterial pressure reflects over low portal pressure. What are hepatic segments? Q They are units of the hepatic parenchyma provided with a branch of hepatic artery and having their own biliary channel (Fig. 5.22). What is the capacity and function of the gallbladder? Q About 50 ml. Its functions being storage and concentration of bile (10 times), regulation of pressure in the biliary system and reduction of alkalinity of hepatic bile. What is Murphy’s sign? Q In acute cholecystitis (inflammation of the gallbladder), pressure in the right hypochondrium causes pain and patient catches breath before zenith of inspiration. Copyrighted material domen and Perineum . Caudate lobe Lateral superior Medial superior _-. Anterior superior Posterior superior Lateral inferior Medial inferior Posterior inferior Fossa for gallbladder Anterior inferior Fig, 5.22 Hepatic segments. What is cholelithiasis? Itrefers to stones in the biliary tract. It is more common in women because pregnancy predisposes to hypercholesterolaemia and disturbances in emptying of the gallbladder. What are the causes of hepatomegaly? Q Hepatomegaly is abnormal enlargement of the liver. It is caused by (i) hepatitis, (ii) fatty infiltration, (iii) biliary obstruction and (iv) malignancy. What is Wilson's disease? Thisis associated with defective copper metabolism. It is characterized by lower levels of serum ceruloplasmin and copper and increase in urinary copper.There is cirrhosis of the liver, marginal pigmentation of the cornea and degeneration of basal ganglia in the central nervous system. What is Addison's disease? In this there is atrophy of the suprarenal cortex (tuberculous infection) and the cortical insufficiency leads to Addison's disease characterized by (i) muscular weakness, (ii) low blood pressure, (ji/) Cutaneous pigmentation, (iv) altered electrolyte balance, and (v) circulatory and renal failure. What is Cushing’s syndrome? An excessive adrenal cortical secretion caused by hyperplasia or tumour Copyrighted material Vv rs seh) results in Cushing's syndrome. it is characterized by (i) obesity, (ii) hirsutism, (iii) impotence (iv) hypogonadism and feminization in males,(v) amenorrhoea and masculinization in females, and (vi) precocious body growth in children. What are Hartmann’s pouch and Phrygian cap? Q Both are anomalies associated with the gallbladder. Hartmann’s pouch is outpouching of the wall of the neck of gallbladder and a site for quiescent gall stone. Phrygian cap is a folded fundus of the gallbladder evident on cholecystogram (Fig. 5.23). Hartmann's pouch Phrygian cap Fig. 5.23 Anomalies of the gallbladder. What is the importance of Calot’s triangle? a Itis a triangle bounded by the common hepatic duct, the cystic duct and the inferior surface of the liver. Through this triangle passes the cystic artery (Fig. 5.24). || Common hepatic duct Cystic node Cystic duct Fig. 5.24 Calot’s triangle. What is biliary dyskinesia? Musculature of the gallbladder wall and the sphincter of Oddi have a reciprocal innervation, i.e. if the gallbladder contracts, the sphincter relaxes. If this mechanism is disrupted it is called biliary dyskinesia. Which conditions mimic the biliary colic? x Biliary colic should be distinguished from (i) the renal colic due to renal stones, Copyrighted material Monnet which may be achieved by lateral radiograph of the abdomen; and (ii) pain of acute appendicitis with subhepatic caecum. What is Saint’s triad? a Itis trio of (j) gall stones, (i) hiatus hernia and (iii) diverticulosis of the colon. How is the word duodenum derived? Q It is derived from the word ‘dodeca dactulous: Do = two, deca = ten, dactulous = fingers, meaning 12 fingers in length. Which part of the duodenum is derived from the foregut and which one from the midgut? Part of the duodenum proximal to the major duodenal papilla is derived from the foregut and distal to it is derived from the midgut (Fig. 5.25). Foregut Major duodenal papilla [= Midgut Fig. 5.25 Embryological background of the duodenum. State precisely the site of opening of the common bile duct. Itopens at the summit of the major duodenal papilla in common with the pancreatic duct, about 9 cm away from the pylorus and on posteromedial wall of the duodenum. Which part of the duodenum forms ‘duodenal cap’ and why? & Proximal half (about 2 cm) of the first part of the duodenum is said to contribute to the formation of duodenal cap due to the following reasons: 1. This part is the only part which has mesoduodenum. 2. It lacks circular folds called plicae circulars. 3. Immediately proximal to it there is a sphincter. Copyrighted material What is ampulla of Vater (also called hepatopancreatic ampulla)? It is the terminal dilated part of the common, bile and pancreatic ducts. What is the ligament of Treitz and what is its function? Itis a band of connective tissue with some muscle fibres in it, from right crus of the diaphragm to the posterior aspect of duodenojejunal flexure. It maintains the duodenojejunal flexure. What is the blood supply of the duodenum? It is furnished by (i) the right gastric artery, (ii) superior and inferior pancreaticoduodenal arteries, (iii) supraduodenal from the hepatic artery and twigs from the gastroduodenal artery (Fig. 5.26). Supraduodenal artery Right gastric artery Superior pancreatico- duodenal artery Inforior pancreatico- duodenal artery Fig. 5.26 Blood supply of the duodenum. Why is pancreas so called? Pan = all, kreas = soft. It is all soft and so it is called pancreas. Uncinate process of the pancreas is sandwiched between which two vessels? a Uncinate process is sandwiched between the aorta behind and the superior mesenteric artery in front (Fig. 5.27). Name an important structure related to the posterior aspect of the neck of the pancreas. Q Formation of the portal vein. Where do pancreatic ducts open? Q Chief pancreatic duct opens in common with common bile duct at the summit of the major duodenal papilla. Accessory pancreatic duct opens Copyrighted material Superior mesenteric — artery Uncinate process Fig. 5.27 Vessels related to the uncinate process. at the summit of minor duodenal papilla placed 2 cm proximal to the major duodenal papilla (Fig. 5.28). Bile duct Minor duodenal _-> Main pancreatic duct papilla . = Accessory pancreatic duct Major duodenal papilla Fig. 5.28 Interior of the second part of duodenum. What does the endocrine component of pancreas secrete? & B cells of islets of Langerhans produce insulin while c,, (or D) cells secrete pancreatic gastrin or possibly serotonin and c, cells secrete glucagon. Name the two veins separated by aditus to lesser sac. Q Portal vein lying in the right free margin of lesser omentum forms an anterior boundary while the inferior vena cava forms its posterior boundary (Fig. 5.29). What are the signs and symptoms of peritonitis? Itis characterized by rigidity, pain, distention of the abdomen, absence of intestinal sound on auscultation and rebound tenderness. Often there is nausea and vomiting. What is Crohn’s disease? Also called regional enteritis; it refers to chronic inflammatory bowel affecting the ileum and/or the colon. Its aetiology is unknown. Copyrighted material y Right free margin of Splenic recess lesser omentum Gastrosplenic ligament Liver Gallbladder Epiploic foramen Lienorenal ligament Splenic arter pI Os Fig. 5.29 Transverse section of the abdomen at the level of epiploic fora- men (IVC, inferior vena cava). What is paralytic ileus? Q It refers to the absence of intestinal peristalsis. It may occur following abdominal surgery. it ischaracterized by abdominal distension, tendemess, absence of bowel sounds, nausea and vomiting. How and where does portal vein formation occur? Q Itis formed by union of the superior mesenteric vein and the splenic vein behind the neck of pancreas at the level of L, vertebra. Enumerate the sites of portal systemic anastomos | foes Gastro-oesophageal junction Left gastric vein Azygos vein Anal canal Superior rectal vein _ Inferior rectal vein At umbilicus (veins of Veins along ligament Body wall veins Sappey) teres 4. Bare area of the colon (veins Mesenteric vein Body wall veins of Retzius) What do the embolic abscesses in the liver reflect over? Qa Involvement of the right lobe reflects over pathology (infective focus) in drainage area of the superior mesenteric vein and that of the left lobe of the liver indicates focus in the area of splenic or inferior mesenteric vein (Fig. 5.30). Copyrighted material Abdomen and Perineum Embolic focus Splenic vein Portal vein Superior mesenteric vein Fig. 5.30 Embolic abscesses in the liver—An anatomical consideration. What are the common manifestations of portal hypertension? (i) Oesophageal varices, (ji) haemorrhaids, (ii) ascites and (iv) caput medusae. What is the normal pressure in the portal vein? 8-12 mm Hg. What is the lower extent of the extension of psoas abscess? In the thigh (behind inguinal ligament), under cover of the psoas fascia What are arcuate ligaments? a Median arcuate ligament extends between the right and the left crus. Medial arcuate is condensation in the upper part of psoas fascia. Lateral arcuate ligament is condensation in the upper part of fascia covering quadratus lumborum (Fig. 5.31). What is the function of psoas major muscle? Flexion at the hip joint (ie. either the trunk bends over the lower limbs or the lower limbs bend over the trunk). It also causes some degree of lateral rotation. Why does the left kidney lie at a higher level than the right? & Thisis because the ascent of the right kidney is restricted by the voluminous liver, while the small-sized spleen permits the ascent of the left kidney to higher level. Copyrighted material image not available Abdomen and Perineum hragm ‘Suprarenal _— Septum Kidney Posterior lamina (Zuckerkandl's fascia) Anterior lamina (Toldt's fascia) Fig. 5.32 Disposition of the renal fascia. Note the septum separating the suprarenal gland from the kidney. Anterior Renal vein Renal artery Ureter ~ Retroureteric branch of renal artery Posterior Fig. 5.33 Relations at the hilum of kidney. Anatomica, they are (/) superior, (i) anterior superior, (iii) anterior inferior, (iv) inferior and (v) posterior (Fig. 5.34). Mention the anatomical sites of constriction along the course of the ureter, a (i) At the pelviureteric junction, (i) where it crosses the brim of the pelvis and (iii) as it enters the wall of the urinary bladder. Copyrighted material Superior Posterior Brodel's line Inferior Fig. Anterior superior Anterior inferior 34 Renal vascular segments. What is the relationship between the ovary and the ureter? & Ureter forms the posterior boundary of the ovarian fossa. Why is the abdominal part of the ureter explored from the lateral side while the pelvic part from the medial side? The abdominal part of the ureter receivesits blood supply from the medial side, it is therefore explored from the lateral side. The pelvic part of the ureter gains its blood supply from the lateral side and so it is approached from the medial side (Fig. 5.35). Abdominal part —| Pelvic part oo Urinary bladder Fig. 5.35 Blood supply of the ureter in abdominal part from medial side and in pelvic part from lateral side. Copyrighted material bdomen and Pi What is the clinical significance of the ureter’s close relation- ship with the cervix uteri? Ureter may be accidently caught in the clamp during hysterectomy. Out of the two ureters which is more vulnerable and close to the vagina? Itis the left ureter, because uterus and cervix both are shifted normally to the right and hence the vagina shifts to the left. Why does the ureteric pain radiate from loin to groin? Q Because the ureter is supplied by T and also the skin over groin. jon and L,.L, supplies the area of loin What is phaeochromocytoma? Itis a tumour involving pheochromocytes in the adrenal medulla and is characterized by (i) palpitation, (ii) excessive sweating, (iii) hypertension, (iv) headache, (v) pallor, (vi) retinitis and (vii) renal vascular changes. All these manifestations are because of excessive secretion of the epinephrine and norepinephrine. What are the indications for sympathectomy? It refers to surgical interruption of the sympathetic pathways, done to relieve intractable pain or to enhance vasodilation in vaso-occlusive diseases. Indications for sympathectomy are (i) Burger's disease, (ii) Raynaud's disease, (iii) claudication and (iv) arteriosclerosis, Where is the mesentery attached to the posterior abdominal wall? Q It is attached from the left (duodenojejunal flexure) of L, vertebra to the right sacroiliac joint (ileocaecal junction) (Fig. 5.36). What are taenia coli? a These are three bands of longitudinal muscle of the large intestine: one on the anterior side and two on the posterior. Out of these the anterior one serves as a guide to the appendix during appendectomy (Fig. 5.37). What is meant by the word caecum? Q The word ‘caecum’ means blind. Copyrighted material (Duodenojejunal flexure) Lett of Lo vertebra /— Line of mesenteric lieocaecal junction attachment Right sacroiliac joint” Fig. 5.36 Line of attachment of mesentery to posterior abdominal wall. Posterior Anterior Caecum Fig. 5.37 Taenia coli. What are the different types of caecum? (i) Conical (infantile) type, (if) quadrate type, (iii) adult type and (iv) exaggerated type Ill (Fig. 5.38). rg Conical type Quadrate type Adult type Exaggerated type Ill (Infantile) (Commonest) Fig. 5.38 Different types of caecum. Copyrighted material Which is the most common position of the appendix? Q Retrocaecal (67%) (Fig. 5.39). domen and P. Postileal Retrocolic Pre-ileal Retrocaecal (67%) lleum Pelvic (21%) Subcaecal Fig. 5.39 Different positions of the appendix. Where is the pain in acute appendicitis felt? Q Initially it appears around the umbilicus or in the epigastrium; however, subsequently it gets localized in the right iliac fossa. What is McBurney’s point? a It indicates the base of the appendix and lies at the junction of lateral ‘one-third and medial two-third of the line joining the anterior superior iliac spine to the umbilicus (Fig. 5.40). Anterior superior iliac spine McBumey's point Inguinal ligament Fig, 540 McBurney’s point. In acute appendicitis what is the site of maximum tenderness? Maximum tenderness in acute appendicitis is at McBurney’s point. Name the artery of the hindgut and at what level does it spring from the aorta. Qa Inferior mesenteric. It springs from the aorta at L, level. Copyrighted material What is the fate of the inferior mesenteric artery? After issuing left colic and sigmoidal branches it continues as the superior rectal artery (Fig. 5.41). Aorta - Inferior mesenteric -—> Left colic artery artery Sigmoidal branches: ‘Superior rectal artery. Fig, 5.41 Fate of the inferior mesenteric artery. Where does the superior rectal artery terminate? Superior rectal artery terminates at S, level, where the pelvic mesocolon ends, i.e. at the rectosigmoid junction by bifurcating into right and left branches. Mention vertebral levels at which the abdominal aorta termi- nates and the inferior vena cava commences. a Abdominal aorta terminates at L, into common iliac arteries and inferior vena cava commences at L, by the confluence of common iliac veins. From where is the last pair of lumbar arteries derived? Small last pair of lumbar arteries arises from iliolumbar arteries or from the median sacral artery. Nane the arteries lying posterior to the inferior vena cava. ( Right lumbar arteries, (/) right renal, (i/) right suprarenal and (Iv) right phrenic arteries. Name the unilateral/unpaired tributaries of the inferior vena cava. Q ( Right inferior phrenic vein, (ji) right suprarenal vein and (ii) right gonadal vein. Copyrighted material Tench From which spinal segments is the lumbosacral trunk derived? Itis derived from part of the ventral ramus of L, and entire L, segment. The sacral segment S, contributes to it at lower level. What forms the pelvic floor? Pelvic floor is formed by levator ani and coccygeus muscles. What are the parts of levator ani muscle? (i) Pubococcygeus, (ii) iliococcygeus and (iii) coccygeus (ischiococeygeus). Puborectalis and levator prostatae are taken as parts of the levator ani muscle (Fig. 5.42). Anterior Fig. 5.42 Parts of the levator ani muscle. What is rectal ampulla? Lower dilated part of the rectum below the middle horizontal (transverse) fold. How does the sigmoid colon differ from the rectum or what are the features at rectosigmoid junction? Presence of (i) taenia coli, (i) sacculations, (iif) appendices epiploicae and (i) pelvic mesocolon distinguishes the sigmoid colon from the rectum. Features at rectosigmoid junction: (i) Bifurcation of the superior rectal artery, (i/) valve of Houston (transverse fold) and (ii) O’Beirne’s sphincter on proctoscopy are additional features associated with the rectosigmoid junction. Copyrighted material State whether rectal ampulla contains faecal matter or not. There are two views: one view is that normally faecal matter lies in the sigmoid colon and not in the rectum, and other view states that faecal matter normally lies above the horizontal fold above rectal ampulla. In any case, rectal ampulla contains faeces only in (i) chronic constipation, (ii) during the act of defaecation and (iii) after death. What is the capacity of urinary bladder? It holds about 400 to 500 ml of urine without overdistension. When does urinary bladder become abdominal? It is abdominal in infants/children, while in adults it rises in the abdomen on distension only. What does the median umbilical ligament represent? Remains of the urachus. What forms the medial umbilical ligamen’ It is formed by the obliterated part of umbilical artery whose patent portion forms the superior vesical artery (Fig. 5.43). Umbilicus Median umbilical ligament (urachus) Urinan bladder Medial umbilical ligament or Superior vesical artery Fig. 5.43 Fate of the urachus and superior vesical artery. What is hiatus hernia? It refers to a diaphragmatic hernia that occurs through the oesophageal hiatus. What is sliding hiatus hernia? Occurring mostly in middle-aged women as a result of progressive weakness of the phrenoesophageal ligament. This leads to the widening of the oesophageal hiatus and the cardiac end, and the fundus of the stomach passes up this hiatus into the thoracic cavity. Copyrighted material Cs What is diaphragmatic breathing? It is a deliberate use of the diaphragm to control respiration, taught to the patients of chronic obstructive pulmonary disease (COPD) to facilitate breathing. What is cave of Retzius? Q Itis also called retropubic space. It lies behind fascia transversalis and pubic symphysis, limited lower down by puboprostatic ligaments. Through this space suprapubic approach to the prostate, urinary bladder and extra peritoneal caesarean section can be achieved (Fig. 5.44). Urinary bladder Cave of Retzius Anterior part Prostate peritoneal membrane of rectal fascia Fig, 5.44 Cave of Retzius and Denonvilliers’ fascia. Q Itis a triangular smooth area on the posterior wall of the urinary bladder with its base formed by the interureteric ridge (Mercier’s bar) and sides formed by Bell's muscles. Trigonal muscle is the continuation of longitudinal musculature of the ureter (Fig. 5.45). What is the trigone of urinary blade: Mercier's bar Ureteric orifice Boll's muscle Fig. 545 Trigone of the urinary bladder. Copyrighted material image not available Ce Urinary bladder Prostate Prostatic utricle Fig. 5.47 Prostatic utricle. What are the routes of surgical approach to the prostate gland? & (i) Suprapubic (through the urinary bladder), (ji) retropubic, (iii) perineal and (iv) transurethral (Fig. 5.48). Suprapubic Retropubic ‘Transurethral Perineal Fig. 5.48 Approaches to the prostate. What is Denonvilliers'’ fascia? a It is connective tissue condensation between urinary bladder, seminal vesicle and prostate anteriorly, and rectum posteriorly in the subperitoneal pelvic space. It has two elements, anterior layer being called the prostato- peritoneal membrane and its posterior layer forms the anterior part of rectal fascia (Fig. 5.44). What is the normal pos of the uterus? a Anteverted and anteflexed. Note: If it is retroverted it may be the cause of infertility. Copyrighted material ioe What is colposcopy? Itis a procedure In which an optical instrument called ‘colposcope’is used for inspection of the cervix and vagina under magnification. It can help in early detection of cervical carcinoma. What information is obtained on hysterosalpingography? & A hysterosalpingogram gives the following information: 1. Tubal patency. 2. Uterine anomalies, fibromyoma, adhesions. 3. Peritubal adhesions can be suspected if the dye is confined to the areas close to the fimbria of uterine tube. What are the degrees of uterine prolapse? What is procidentia? 1. Uterine prolapse of the first degree is the descent of cervix in the vagina. 2. Second degree is when the cervix protrudes outside the vaginal canal. Procidentia refers to the condition in which the entire uterus is pushed outside the vaginal vestibule. What are the indications for pelvic laparoscopy? Indications for pelvic laparoscopy are (i) unexplained infertility, (ii) ectopic pregnancy, (iii) diagnosis/management of pelvic mass/pain, (iv) retrieval of ova for in vitro fertilization (IVF) and (v) evaluation of pelvic peritonitis. What are the indications for hysterectomy? Itis done in the following: (i) fibroid uterus, (ii) carcinoma of the uterus, (ii) chronic recurrent pelvic disease, (iv) uterine haemorrhage and (v) severe recurrent endometrial hyperplasia. PERINEUM What is episiotomy? a Itis incising the perineum in order to enlarge the birth canal.The superficial perineal muscles are incised. The incision is given in posterolateral direction. This prevents perineal tear, involving perineal body which is likely to be followed by uterine prolapse. Copyrighted material noun [141 Abdomen and P. What is obstetrical conjugate? Itis also called diagonal conjugate.|t refers to the distance from the lower border of pubic symphysis to the sacral promontory. It is important in clinical pelvimetry so as to have an assessment about adequacy of the pelvis for normal delivery? Where is the ischiorectal fossa located? It is a potential space located between obturator internus laterally and levator ani and anal canal medially (Fig. 5.49). Levator ani Longitudinal 1 “|||, Supra- muscle — 1 if tegmental ' space Circular —} 1 ( muscle ‘ -\Fascia lunata t Obturator = | fascia Internal _| + Obturator o ce] sphincter i internus | Alcock's canal Ischium Sphincter - ‘ani exterus 'Schiorectal space Fig, 5.49 Fascia lunata and suprategmental space—A coronal section. What is Alcock’s canal (pudendal canal)? It lies between obturator fascia and fascia lunata and contains pudendal nerve and internal pudendal vessels. What is pilonidal sinus? An obstruction of hair follicles in the natal cleft is associated with ingrowths of hair. It may be enhanced by prolonged sitting or rolling action of obese buttock. The sinus is lined by epithelium and its opening is placed 4 to 8 cm above the anus. Copyrighted material What is Goodsall-Salmon’s law? It is to know the course of fistulous tract and location of the primary ‘opening in fistula in ano. 1. Fistula with the external opening located posterior to the imaginary line passing transversely through the anal orifice, usually having the internal opening in posterior mid-line and the tract is curved. 2. The tract is straight and in line with the secondary opening if the external (secondary) opening is anterior to the transverse line. What are the contents of ischiorectal fossa? a (i) Fat (loculated), (j/) inferior haemorrhoidal (rectal) vessels, and (iii) inferior haemorrhoidal (rectal) nerve and its branches. What is fascia lunata? Q Itarches over ischiorectal fat contributing to the roof and lateral wall of the ischiorectal fossa. It blends with the inferior fascia of the pelvic diaphragm superiorly and fuses with the falciform process of the sacrotuberous ligament inferiorly (Fig. 5.49). What is the functional significance of ischiorectal fossa? a It permits distension of the anal canal during defaecation and after this by elastic recoil the fat brings the anal canal back to its original state. Why are ischiorectal abscesses painful? Q Because ischiorectal fossa is divided by numerous fibrous septa having rich nerve supply. These nerve fibres are stretched by expanding ischiorectal abscess causing severe pain. What is hiatus of Schwalbe? A gap that is present occasionally between the obturator fascia and the tendinous arch of origin of levator ani muscle. It communicates suprategmental space with pelvic cavity. What is suprategmental space? It is a well-marked space near the apex of the ischiorectal fossa and tegmental fascia (fascia at the apex). It is devoid of fat (Fig. 5.49). Which is the voluntary sphincter for anal canal? Q Sphincter ani externus. Copyrighted material CE What is Sim’s position? It is the left lateral position given to the patient in which per rectal examination can be done. What is the nerve supply of sphincter ani externus? Q It is supplied by the inferior rectal branch of pudendal nerve and the perineal branch of fourth sacral nerve. Name the part of sphincter ani externus which has bony anchorage. Superficial part of sphincter ani externus extends from the tip of coccyx to the perineal body. What is colostomy? Itis creating an artificial opening in the large bowel to divertits contents to the exterior either temporarily or permanently. The site is either transverse or pelvic colon. Colostomy bag is provided to collect the contents. What is rectocoele? Protrusion of the rectum and posterior vaginal wall into the vagina. It occurs because of weakness of the pelvic floor and the vaginal wall due to old age, childbearing or after surgery. What is the function of perineal body and anococcygeal raphe? Q To support pelvic viscera such as the uterus, rectum and anal canal; evidenced by prolapse of these organs with the rupture of these elements. What is the clinical importance of white line of Hilton? Qa Lesionsinvolving the anal canal below this line are painful (being supplied by somatic nerves) and above this line are painless (being supplied by visceral fibres) (Fig. 5.50). Where do lymphatics from the lower half of the anal canal terminate? Q In the superficial inguinal group of lymph nodes. Copyrighted material Sphincter ani internus Deep Parts of external a Subcutaneous —~ Fig. 5.50 White line of Hilton, and external and intemal anal sphincters. Visceral nerve Somatic nerve White line What is the importance of the attachment of the membranous layer of superficial fascia? Itis important for determining the course of extravasated urine in rupture of the urethra. What is perineal membrane (also called inferior fascia of uro- genital diaphragm)? Itis a fibrous membrane stretching between conjoint ischiopubic rami. Mention the attachment of Colles’ fas: a Itis attached to the conjoint ischiopubic rami and the posterior margin of perineal membrane. In the thigh, it is attached horizontally to the fascia lata below the inguinal ligament (Fig. 5.51). N Anterior superior iliac spine Fig. 5.51 Line of attachment of Colles’ fascia. On straddle injury to the urethra where does extravasated urine escape? A straddle injury to the urethra allows the extravasated urine to fill the superficial perineal pouch, loose tissue of the scrotum and penis. It then ascends up into the anterior abdominal wall deep into the membranous Copyrighted material EE layer of superficial fascia (Scarpa’s fascia), passes into the upper part of thighs up to Holden's line (where Scarpa's fascia fuses with fascia lata). What is transmitted through the gap between pubic symphysis and transverse perineal ligament? Deep dorsal vein and dorsal nerve of the penis or clitoris. Enumerate the structures piercing the perineal membrane. % In males: (i) urethra, (i/) ducts of bulbourethral glands, (iii) artery and nerve to the bulb, (iv) urethral artery, (v) deep and dorsal arteries of the penis, and (vi) posterior scrotal vessels and nerves. In females: (i) urethra, (ii) vagina, (iii) artery and nerve to the bulb of vestibule, (vi) deep and dorsal artery of the clitoris, and (v) posterior labial nerves and arteries (Fig. 5.52). Dorsal vein Dorsal nerve Dorsal vein of penis of clitoris ) - of clitoris pesp Dorsal ] 2 artery of Dorsal artery Urethra ey Le clitoris © \ of clitoris Urethral a eee [5 Urs Perineal artery : my Artery to fmemnBFanS Arlery to bulb of Perineal bulb vestibule membrane of penis Vagina Duct of Posterior scrotal Posterior labial bulbourethral vessels vessels and nerves gland and nerves (Female) (Male) Fig. 5.52 Structures piercing the perineal membrane. What is transverse ligament of perineum? It is the thickened anterior border of perineal membrane (Fig. 5.52). In which perineal pouch the bulbourethral glands are located and where do their ducts end? The glands are located in the deep perineal pouch while the ducts pierce the perineal membrane to enter the superficial perineal pouch and finally end up in the urethra (Fig. 5.53). Why is the perineal membrane weaker in females? Because of the vaginal passage. Copyrighted material Urinary bladder Seminal vesicle Prostate Deep perineal pouch Fig. 5.53 Bulbourethral glands with their ducts. What is the peculiarity of the superficial fascia of scrotum? Itis modified into dartos muscle which is responsible for rugosity of the scrotal skin. What is the action of bulbospengiosus muscle? Q 1. Evacuates the last drop of urine and semen. 2. Possibly aids the erection of penis. What is the function of transversus perinei superficialis? They fix the perineal body which Is crucially important for effective contraction of sphincter ani externus as well as bulbospongiosus. Which is the shortest and which is the most distensible part of the urethra? Q The membranous part is the shortest (half an inch) and prostatic part is the most distensible part of the urethra. How is the course of male urethra? Like letter ‘S Why is it essential to know the anatomy of urethra? For the passage of catheter/instrument, viz. urethroscope. Name the voluntary sphincter controlling the act of micturition. Qa Sphincter urethrae. What are the causes of urinary incontinence? It refers to the involuntary passage of urine. Its causes are (i) neurogenic Copyrighted material a bladder, i.e. brain and/or spinal cord lesions, (ii) multiple sclerosis, (iii) trauma, (iv) obstructive lesion in the lower urinary tract and (v) tumour or calculus in the bladder. What is Buck’s fascia? Trace its continuity. Q Itis the penile fascia surrounding three erectile elements of the penis and is continuous with Colles’ and Scarpa's fasciae (Fig. 5.54). Corpora cavernosa Penile fascia Corpus spongiosum Fig. 5.54 Buck’s fascia surrounding erectile elements of the penis. What is the mechanism of erection of penis? Itis by increased blood flow following stimulation of S,_, parasympathetic fibres (8, ,) and possibly sympathetic fibres from T, -L,.S,_, are controlled by the limbic system and the hypothalamus receiving sexual stimuli from visual, auditory and tactile sensations, etc. What is the difference between penis and phallus? Q Phallusis an elongated genital tubercle with urethral groove on the ventral side. Penis is the male genital organ with three erectile elements and the urethra incorporated within it. How does the prepuce separate from the glans? By the process of desquamation of the epithelium. What is phimosis? Congenital or acquired narrow opening of the prepuce is called phimosis. Name the lymph node draining glans penis. Q Lymph node of Cloquet (lodged in the femoral canal). Copyrighted material OSTEOLOGY How would you differentiate hetween a male and a female pelvis? Features of the female pelvis 1, Greater sciatic notch is wider. 2. Rounded and wide subpubic angle. 3. Diameter of the acetabulum is less than the distance from its anterior margin to the pubic symphysis. . Superior articular facet of S, is equal to or less than alae. . Nearly circular inlet of the pelvis. . Abruptly curved sacrum. . Pre-auricular sulcus is present. . Bones are delicate. 9. Ilia are more vertical. ON AuE Features of the male pelvis . Greater sciatic notch is narrower. 2. Acute subpubic angle. 3. Acetabular diameter is equal to the distance between pubic symphysis and its anterior margin. 4. Superior articular facet of S, is wider than the alae. 5. Everted ischiopubic rami. 6. Heart-shaped inlet of the pelvis. 7. 8. . Sacrum is evenly curved. . Intercristal distance is more. ate the points of sexual dimorphism in the sacrum. & 1. Superior articular facet of S, exceeds the width of ala in male. 2. Sacral index, i.e. breadth/length x 100 = 105 in males and up to 115 in females. 3. Auricular surface extends up to S, in females and midway along S. in males. 4. Ventral/pelvic surface of the sacrum shows an abrupt curvature in female. 3 What represents the costal element in the sacrum? In the sacrum, costal element is represented by part of ala or lateral mass and bars between pelvic sacral foramina. Copyrighted material ice econ ae a Name the structures related to the ala of sacrum. Q From medial to lateral sides these are (/) lumbosacral trunk, (ii) iliolumbar artery and (iii) obturator nerve (Fig. 5.55). Lumbosacral 4 lliclumbar artery liacus Obturator nerve Root of pelvic mesocolon Piriformis Median sacral artery Coceygeus Fig. 5.55 Structures related to the ala of sacrum. Copyrighted material Head and Neck Enumerate the layers of the scalp. a (i) Skin, (i) dense connective tissue, (iii) epicranial aponeurosis, (iv) loose areolar tissue and (v) pericranium (Fig. 6.1). ‘Skin Dense connective tissue Galea aponeurotica Loose areolar tissue Pericranium Fig, 6.1 Layers of the scalp. Why are sebaceous cysts and seborrhoea more frequently associated with the scalp? Because the skin of the scalp is hairy and contains a large number of sebaceous glands associated with the hair follicles. Why wounds of the scalp bleed profusely? Q This is because of rich vascular supply to the scalp with rich anastomosis between branches of the internal carotid artery (ICA) and the external carotid artery (ECA). The injured vessels are prevented from retracting effectively by the dense connective tissue of the scalp. Bleeding can be effectively stopped by applying firm pressure against the underlying bone. Copyrighted materia Head and Neck [151 Name the muscle of the scalp and mention its nerve supply and action. The muscle of the scalp is occipitofrontalis. It is supplied by posterior auricular and temporal branches of the facial nerve. Its action causes horizontal wrinkling of the forehead. Why s It is because the dense connective tissue (one of the layers of the scalp) has fibrous strands. It is unyielding in nature and this creates pressure on the nerves running across and hence the pain. Ip swellings are painful? Q Which is the dangerous layer of the scalp? Q The layer of loose connective tissue (between galea aponeurotica and pericranium). Why do injuries of the scalp cause black eye? q After injury, blood accumulates in the subaponeurotic space and then gravitates down in the loose connective tissue around eye. Note: Frontal belly of the occipitofrontalis does not have a bony attachment. What is an indirect black eye? Black eye can be either because of a direct blow to the eye leading to a direct black eye or alternatively due to an injury to the scalp, with the accumulation of blood in the subaponeurotic space (under epicranial aponeurosis) and later on blood gravitates under frontalis to the upper eyelid followed by the lower eyelid, leading to ‘indirect black eye! It appears usually 1 to 2 days following the injury. What is safety valve haematoma? a With fractures involving the cranial bone, initially the blood accumulates in the subaponeurotic space (loose areolar tissue). Only when this space is full, the blood trickles through the fracture line into cranial cavity to make the signs of cerebral compression evident. Thus, the layer of loose connective tissue serves as 4 safety valve initially by preventing cerebral compression (Fig. 6.2). What is a traumatic cephal-hydrocoele? Itrefers to a scalp swelling following an injury to the skull, causing fracture of the skull, damage to the meninges with the escape of cerebrospinal fluid (CSF). Galea aponeurotica Subaponeurotic space Haematoma Brain Fig. 6.2 Safety valve haematoma. What is caput succedaneum? It refers to a localized pitting oedema on the scalp of a newborn baby usually formed during labour as a result of circular pressure of the cervix on the fetal occiput. What is the cause of neck rigidity in meningitis? It is due to spasm of the extensors caused by irritation of nerve roots traversing through the infected subarachnoid space. What is cisternal puncture? It is withdrawing CSF by inserting needle in the cerebellomedullary cistern. What is the peculiarity of emissary veins and what is the merit and demerit of this peculiarity? a The peculiarity of emissary veins is that they are devoid of valves and hence blood can flow in either direction. Merit —_ Inhigh intracranial tension blood can flow outside, saving nervous tissue. Demerit — Infection of the scalp can travel in and cause meningitis (Fig. 6.3). Extracranial vein Emissary vein Dural venous: sinus Fig. 6.3 Emissary vein. Copyrighted material image not available image not available PEE EEEE SEC Fig. 6.4 Dangerous area of the face. cavernous sinus. Therefore, lesions of this area can carry infection to the cavernous sinus leading to its thrombosis (Fig. 6.4). What is the clinical significance of the colour of conjunctiva? 1. Yellow discolouration of the bulbar conjunctiva indicates jaundice. 2. Paleness of the palpebral conjunctiva speaks of anaemia. What is a stye or hordeolum? It is an acute inflammation of the Zeis gland causing a painful swelling which points near the base of the cilium. What is a panda bear sign? A leakage of blood into the periorbital tissue due to damage to the anterior group of dural venous sinuses is called panda bear sign. In this, subconjunctival haemorrhage is absent which is found in direct ocular trauma. What is the effect of carotid cavernous aneurysm on the eye- ball? The communication between the internal carotid artery and the cavernous sinus is called carotid cavernous aneurysm and leads to a pulsating and bulging eyeball. Which cervical spinal segment does not have cutaneous distribution? a First cervical segment (C,). Copyrighted material Name the muscles, glands and spaces enclosed by investing layer of the deep cervical fascia. Q Muscles. — Sternocleidomastoid and trapezius Glands. = — Parotid and submandibular Spaces — Suprasternal and supraclavicular imal gland. a Lacrimatory nucleusin pons —» nervous intermedius —» geniculate ganglion —> greater superficial petrosal nerve —> nerve of the pterygoid canal > pterygopalatine ganglion > zygomatic nerve >» zygomaticotemporal nerve — lacrimal nerve — lacrimal gland (Fig. 6.5). ‘Trace the secretomotor pathway of the Iz Lacrimatory nucleus Nervous intermedius Geniculate ganglion Greater superficial petrosal nerve yp Sygomatic nerve Zygomaticotemporal nerve Pons \ Pterygopalatine ganglion Lacrimal gland Deep petrosal nerve Fig. 6.5 Secretomotor pathway of the lacrimal gland. What is stylomandibular ligament? Q Condensation of the investing layer of deep cervical fascia between styloid process and angle of mandible. What is the function of prevertebral fascia? Q It provides the basis on which the mobile viscera of the neck can move (Fig. 6.6). What is the significance of ligaments of Berry? Q These are condensations of the pretracheal fascia between thyroid gland and laryngeal cartilages and serve to support the thyroid gland and that is why thyroid swelling moves up during swallowing. Copyrighted material image not available image not available image not available image not available image not available image not available image not available image not available image not available image not available image not available image not available image not available aa You have either reached a page that is unavailable for viewing or reached your viewing limit for this book. image not available image not available aa You have either reached a page that is unavailable for viewing or reached your viewing limit for this book. image not available aa You have either reached a page that is unavailable for viewing or reached your viewing limit for this book. aa You have either reached a page that is unavailable for viewing or reached your viewing limit for this book. image not available image not available aa You have either reached a page that is unavailable for viewing or reached your viewing limit for this book. image not available aa You have either reached a page that is unavailable for viewing or reached your viewing limit for this book. aa You have either reached a page that is unavailable for viewing or reached your viewing limit for this book. aa You have either reached a page that is unavailable for viewing or reached your viewing limit for this book. aa You have either reached a page that is unavailable for viewing or reached your viewing limit for this book. aa You have either reached a page that is unavailable for viewing or reached your viewing limit for this book. aa You have either reached a page that is unavailable for viewing or reached your viewing limit for this book. aa You have either reached a page that is unavailable for viewing or reached your viewing limit for this book. aa You have either reached a page that is unavailable for viewing or reached your viewing limit for this book. aa You have either reached a page that is unavailable for viewing or reached your viewing limit for this book. aa You have either reached a page that is unavailable for viewing or reached your viewing limit for this book. aa You have either reached a page that is unavailable for viewing or reached your viewing limit for this book. aa You have either reached a page that is unavailable for viewing or reached your viewing limit for this book. aa You have either reached a page that is unavailable for viewing or reached your viewing limit for this book. aa You have either reached a page that is unavailable for viewing or reached your viewing limit for this book. aa You have either reached a page that is unavailable for viewing or reached your viewing limit for this book. aa You have either reached a page that is unavailable for viewing or reached your viewing limit for this book. aa You have either reached a page that is unavailable for viewing or reached your viewing limit for this book. aa You have either reached a page that is unavailable for viewing or reached your viewing limit for this book. aa You have either reached a page that is unavailable for viewing or reached your viewing limit for this book. aa You have either reached a page that is unavailable for viewing or reached your viewing limit for this book. aa You have either reached a page that is unavailable for viewing or reached your viewing limit for this book. aa You have either reached a page that is unavailable for viewing or reached your viewing limit for this book. aa You have either reached a page that is unavailable for viewing or reached your viewing limit for this book. aa You have either reached a page that is unavailable for viewing or reached your viewing limit for this book. aa You have either reached a page that is unavailable for viewing or reached your viewing limit for this book. aa You have either reached a page that is unavailable for viewing or reached your viewing limit for this book. aa You have either reached a page that is unavailable for viewing or reached your viewing limit for this book. aa You have either reached a page that is unavailable for viewing or reached your viewing limit for this book. aa You have either reached a page that is unavailable for viewing or reached your viewing limit for this book. aa You have either reached a page that is unavailable for viewing or reached your viewing limit for this book. aa You have either reached a page that is unavailable for viewing or reached your viewing limit for this book. aa You have either reached a page that is unavailable for viewing or reached your viewing limit for this book. aa You have either reached a page that is unavailable for viewing or reached your viewing limit for this book. image not available image not available aa You have either reached a page that is unavailable for viewing or reached your viewing limit for this book. aa You have either reached a page that is unavailable for viewing or reached your viewing limit for this book. aa You have either reached a page that is unavailable for viewing or reached your viewing limit for this book. aa You have either reached a page that is unavailable for viewing or reached your viewing limit for this book. aa You have either reached a page that is unavailable for viewing or reached your viewing limit for this book. aa You have either reached a page that is unavailable for viewing or reached your viewing limit for this book. aa You have either reached a page that is unavailable for viewing or reached your viewing limit for this book. aa You have either reached a page that is unavailable for viewing or reached your viewing limit for this book. aa You have either reached a page that is unavailable for viewing or reached your viewing limit for this book. aa You have either reached a page that is unavailable for viewing or reached your viewing limit for this book. aa You have either reached a page that is unavailable for viewing or reached your viewing limit for this book. aa You have either reached a page that is unavailable for viewing or reached your viewing limit for this book. aa You have either reached a page that is unavailable for viewing or reached your viewing limit for this book. aa You have either reached a page that is unavailable for viewing or reached your viewing limit for this book. aa You have either reached a page that is unavailable for viewing or reached your viewing limit for this book. aa You have either reached a page that is unavailable for viewing or reached your viewing limit for this book. aa You have either reached a page that is unavailable for viewing or reached your viewing limit for this book. aa You have either reached a page that is unavailable for viewing or reached your viewing limit for this book. aa You have either reached a page that is unavailable for viewing or reached your viewing limit for this book. aa You have either reached a page that is unavailable for viewing or reached your viewing limit for this book. aa You have either reached a page that is unavailable for viewing or reached your viewing limit for this book. aa You have either reached a page that is unavailable for viewing or reached your viewing limit for this book. aa You have either reached a page that is unavailable for viewing or reached your viewing limit for this book. aa You have either reached a page that is unavailable for viewing or reached your viewing limit for this book. aa You have either reached a page that is unavailable for viewing or reached your viewing limit for this book. aa You have either reached a page that is unavailable for viewing or reached your viewing limit for this book. aa You have either reached a page that is unavailable for viewing or reached your viewing limit for this book. aa You have either reached a page that is unavailable for viewing or reached your viewing limit for this book. aa You have either reached a page that is unavailable for viewing or reached your viewing limit for this book. aa You have either reached a page that is unavailable for viewing or reached your viewing limit for this book. aa You have either reached a page that is unavailable for viewing or reached your viewing limit for this book. aa You have either reached a page that is unavailable for viewing or reached your viewing limit for this book. aa You have either reached a page that is unavailable for viewing or reached your viewing limit for this book. aa You have either reached a page that is unavailable for viewing or reached your viewing limit for this book. aa You have either reached a page that is unavailable for viewing or reached your viewing limit for this book. aa You have either reached a page that is unavailable for viewing or reached your viewing limit for this book. aa You have either reached a page that is unavailable for viewing or reached your viewing limit for this book. aa You have either reached a page that is unavailable for viewing or reached your viewing limit for this book. aa You have either reached a page that is unavailable for viewing or reached your viewing limit for this book. aa You have either reached a page that is unavailable for viewing or reached your viewing limit for this book. aa You have either reached a page that is unavailable for viewing or reached your viewing limit for this book. aa You have either reached a page that is unavailable for viewing or reached your viewing limit for this book. aa You have either reached a page that is unavailable for viewing or reached your viewing limit for this book. aa You have either reached a page that is unavailable for viewing or reached your viewing limit for this book. aa You have either reached a page that is unavailable for viewing or reached your viewing limit for this book. aa You have either reached a page that is unavailable for viewing or reached your viewing limit for this book. aa You have either reached a page that is unavailable for viewing or reached your viewing limit for this book. aa You have either reached a page that is unavailable for viewing or reached your viewing limit for this book. aa You have either reached a page that is unavailable for viewing or reached your viewing limit for this book. aa You have either reached a page that is unavailable for viewing or reached your viewing limit for this book. aa You have either reached a page that is unavailable for viewing or reached your viewing limit for this book. aa You have either reached a page that is unavailable for viewing or reached your viewing limit for this book. aa You have either reached a page that is unavailable for viewing or reached your viewing limit for this book. aa You have either reached a page that is unavailable for viewing or reached your viewing limit for this book. aa You have either reached a page that is unavailable for viewing or reached your viewing limit for this book. aa You have either reached a page that is unavailable for viewing or reached your viewing limit for this book. aa You have either reached a page that is unavailable for viewing or reached your viewing limit for this book. aa You have either reached a page that is unavailable for viewing or reached your viewing limit for this book. aa You have either reached a page that is unavailable for viewing or reached your viewing limit for this book. aa You have either reached a page that is unavailable for viewing or reached your viewing limit for this book. aa You have either reached a page that is unavailable for viewing or reached your viewing limit for this book. aa You have either reached a page that is unavailable for viewing or reached your viewing limit for this book. aa You have either reached a page that is unavailable for viewing or reached your viewing limit for this book. aa You have either reached a page that is unavailable for viewing or reached your viewing limit for this book. aa You have either reached a page that is unavailable for viewing or reached your viewing limit for this book. aa You have either reached a page that is unavailable for viewing or reached your viewing limit for this book. aa You have either reached a page that is unavailable for viewing or reached your viewing limit for this book. aa You have either reached a page that is unavailable for viewing or reached your viewing limit for this book. aa You have either reached a page that is unavailable for viewing or reached your viewing limit for this book. aa You have either reached a page that is unavailable for viewing or reached your viewing limit for this book. aa You have either reached a page that is unavailable for viewing or reached your viewing limit for this book. aa You have either reached a page that is unavailable for viewing or reached your viewing limit for this book. aa You have either reached a page that is unavailable for viewing or reached your viewing limit for this book. aa You have either reached a page that is unavailable for viewing or reached your viewing limit for this book. aa You have either reached a page that is unavailable for viewing or reached your viewing limit for this book. aa You have either reached a page that is unavailable for viewing or reached your viewing limit for this book. aa You have either reached a page that is unavailable for viewing or reached your viewing limit for this book. aa You have either reached a page that is unavailable for viewing or reached your viewing limit for this book. aa You have either reached a page that is unavailable for viewing or reached your viewing limit for this book. aa You have either reached a page that is unavailable for viewing or reached your viewing limit for this book. aa You have either reached a page that is unavailable for viewing or reached your viewing limit for this book. aa You have either reached a page that is unavailable for viewing or reached your viewing limit for this book. aa You have either reached a page that is unavailable for viewing or reached your viewing limit for this book. aa You have either reached a page that is unavailable for viewing or reached your viewing limit for this book. aa You have either reached a page that is unavailable for viewing or reached your viewing limit for this book. aa You have either reached a page that is unavailable for viewing or reached your viewing limit for this book. aa You have either reached a page that is unavailable for viewing or reached your viewing limit for this book. aa You have either reached a page that is unavailable for viewing or reached your viewing limit for this book. aa You have either reached a page that is unavailable for viewing or reached your viewing limit for this book. aa You have either reached a page that is unavailable for viewing or reached your viewing limit for this book. aa You have either reached a page that is unavailable for viewing or reached your viewing limit for this book. aa You have either reached a page that is unavailable for viewing or reached your viewing limit for this book. aa You have either reached a page that is unavailable for viewing or reached your viewing limit for this book. aa You have either reached a page that is unavailable for viewing or reached your viewing limit for this book. aa You have either reached a page that is unavailable for viewing or reached your viewing limit for this book. aa You have either reached a page that is unavailable for viewing or reached your viewing limit for this book. aa You have either reached a page that is unavailable for viewing or reached your viewing limit for this book. aa You have either reached a page that is unavailable for viewing or reached your viewing limit for this book. aa You have either reached a page that is unavailable for viewing or reached your viewing limit for this book.

You might also like